Identity proof using Stoke's Theorem

Click For Summary
The discussion centers on applying Stokes' Theorem to demonstrate an identity involving a scalar field and its gradient. The user initially struggles with evaluating the expression (\nabla f) × d\vec S and seeks guidance on manipulating the integrals. By defining a vector function \textbf{F} as f\textbf{c}, where \textbf{c} is a constant vector, they successfully derive an equation involving surface and line integrals. The conclusion drawn is that since the resulting equation holds for any constant vector \textbf{c}, the expression inside must equal zero. The conversation emphasizes the validity of pulling constant vectors out of integrals in this context.
pretzsp
Messages
3
Reaction score
0

Homework Statement



Show using Stoke's Theorem that
gif.latex?\bg_white%20\iint_S%20(\nabla%20f)\times%20d\vec%20S%20=%20-\oint_C%20fd%20\vec%20r.gif


S is an open surface with boundary C (a space curve). f(\vec r) is a scalar field.

Homework Equations



Stoke's theorem \iint_S (\nabla\times \vec F) \cdot d\vec S = \int_C F \cdot d\vec r

The Attempt at a Solution



Thus far I've tried evaluating (\nabla f)\times d\vec S by taking components in various ways and then trying to force it to become something that I can take the curl of... with no success at all. I'm completely stumped now.
 
Physics news on Phys.org
Try applying Stokes' theorem to the vector function \textbf{F}\equiv f\textbf{c}, where \textbf{c} is any constant vector.:wink:
 
Well, what I get here is

\iint_S \vec c\cdot\nabla f \times d\vec S = -\oint \vec c \cdot fd\vec r

And I'm not entirely sure whether I can just "cancel" the c dot product...

(should be a vector c there, by the way)
 
Last edited:
Well, since \textbf{c} is a constant vector, you can certainly pull it out of the integrals and say

\vec c\cdot\left(\iint_S \nabla f \times d\vec S\right) = \vec c\cdot\left(-\oint fd\vec r\right)

\implies \vec c\cdot \left(\iint_S \nabla f \times d\vec S+\oint fd\vec r\right)=0

And since this is true for arbitrary, constant \vec{c}, what must you conclude?
 
Yeah, those last steps were fine for me, I just wasn't sure if that even with c a constant vector that it was "allowed" to just pull it out like so. Thanks for the help.
 
Question: A clock's minute hand has length 4 and its hour hand has length 3. What is the distance between the tips at the moment when it is increasing most rapidly?(Putnam Exam Question) Answer: Making assumption that both the hands moves at constant angular velocities, the answer is ## \sqrt{7} .## But don't you think this assumption is somewhat doubtful and wrong?

Similar threads

  • · Replies 4 ·
Replies
4
Views
2K
  • · Replies 26 ·
Replies
26
Views
3K
  • · Replies 9 ·
Replies
9
Views
2K
  • · Replies 6 ·
Replies
6
Views
2K
  • · Replies 4 ·
Replies
4
Views
2K
  • · Replies 6 ·
Replies
6
Views
3K
Replies
3
Views
2K
Replies
6
Views
3K
  • · Replies 7 ·
Replies
7
Views
2K
  • · Replies 6 ·
Replies
6
Views
2K